LSAT and Law School Admissions Forum

Get expert LSAT preparation and law school admissions advice from PowerScore Test Preparation.

User avatar
 Dave Killoran
PowerScore Staff
  • PowerScore Staff
  • Posts: 5852
  • Joined: Mar 25, 2011
|
#26440
Complete Question Explanation
(The complete setup for this game can be found here: lsat/viewtopic.php?t=8639)

The correct answer choice is (B)

Remember, Must Be True EXCEPT is the same as Not Necessarily True. The condition in the question stem produces the following diagram:
December 2000_game#1_M12_L1_explanations_game#2_#5_diagram_1.png
H, J, and Q must be ranked first, second, and third, respectively. Answer choices (A), (D), and (E) can thereby be eliminated. The highest V can be ranked is fifth and that eliminates answer choice (C). Since P could be fourth, it follows that L does not have to be fourth and answer choice (B) is correct.


For most test takers the appearance of a Pure Sequencing game is cause for celebration. Remember, Pure Sequencing games can be easily identified since the majority of rules are relative in nature. As you work through this book, keep in mind that part of your task is to learn how to quickly identify each game type.
You do not have the required permissions to view the files attached to this post.
 Marce
  • Posts: 27
  • Joined: Sep 05, 2016
|
#32000
HOW CAN YOU TELL THAT Q MUST BE PLACED AFTER J IN THE SEQUENCING DIAGRAM, VS. DRAWING A SEQUENCE LINE IN FRONT OF L, SUCH AS:
Q ------ L THEN THE DOUBLE BRANCHES FOR S & V
 Jon Denning
PowerScore Staff
  • PowerScore Staff
  • Posts: 904
  • Joined: Apr 11, 2011
|
#32035
Hi Marce,

Thanks for the question, and welcome to the Forum!

There are a lot of moving pieces in this game (five rules, four of which are all about sequences and often contain multiple variables), so I can understand how some confusion might arise! I'll walk you through the relevant pieces below, and hopefully that helps to clear things up :)

First, the relationship between Q and J is given directly by the second rule: J is more popular than Q. So we always have the following connection in our diagram (even if other variables get added to it):

..... J :longline: Q

From there in our setup Q branches to P and S (from rule 4), H splits to J and L, and so on as seen in the main diagram discussion here: lsat/viewtopic.php?t=8639

For question #5 however we're given some additional information to consider: Q is now ahead of L (Q :longline: L)

So let's think about what that's going to do to our original sequence. Initially H could be followed immediately by either J or L, and then we continued branching from there (L to S or V, J to Q which then could split...), but if we move L behind Q then H must be followed right away by J. After J would come Q, then L and P, with L splitting as usual to S and V.

That means the first three variables are fixed:

..... H :longline: J :longline: Q in 1, 2, 3

Essentially what question 5 does is remove some of the uncertainty near the beginning of the sequence by moving a possible early variable, L, further back behind Q.

So that's where the H :longline: J :longline: Q sequence comes from: it's the same sequence from the original setup (from the rules) if L is placed later in the diagram.

I hope that helps! If you have any questions though (or if perhaps this didn't completely clear it up) please let me know!
 Marce
  • Posts: 27
  • Joined: Sep 05, 2016
|
#32089
Hi Jon,

Thanks for the detailed explanation, it did help; however Im still stumped on S and V branching off of L in the new digram for question 5. The original rules have S branching off from Q, please explain this for me.
 Adam Tyson
PowerScore Staff
  • PowerScore Staff
  • Posts: 5153
  • Joined: Apr 14, 2011
|
#32097
Check the original rules again, Marce, and you'll see that S branches off from both L and Q. Q is before P and S, and L is before S and V. In this new diagram, for this local question, Q is also before L, so branching S off from L satisfies both of those rules about S - it is after L, per the rules, and since L is after Q, S is also after Q, per the rules. It wouldn't be efficient, or necessary, to show S coming directly off of Q because it still has to be after L. This diagram keeps it simple and clean.

Re-read those rules, plus the additional restrictions set up by this question's stem, and you'll see that this diagram does exactly what's required to comply with all of it.
 Marce
  • Posts: 27
  • Joined: Sep 05, 2016
|
#32287
Thanks Adam makes more sense, this is so complex to me; I have to practice more to get aclimated with LOGIC GAMES! :0
 jessamynlockard
  • Posts: 42
  • Joined: Jan 15, 2018
|
#44493
Since we're told S isn't 7th, should we draw a circle around our S on the diagram?
 Shannon Parker
PowerScore Staff
  • PowerScore Staff
  • Posts: 147
  • Joined: Jun 08, 2016
|
#44526
jessamynlockard wrote:Since we're told S isn't 7th, should we draw a circle around our S on the diagram?
That would be one way to do it. But yes it would be important to have an annotation or marking that you would be comfortable with to remind you that S cannot be 7th.
 brittanychanel
  • Posts: 2
  • Joined: Mar 10, 2019
|
#63271
Greetings!

My confusion is based in the following:

How do we know P + L's relationship? Why couldn't the order potentially be:

H, J, Q, L, P, V, S?

Unless I am reading the diagram wrong, I still have confusion here. I ended up choosing "C" as an answer... any help is appreciated!

Thanks
 Jay Donnell
PowerScore Staff
  • PowerScore Staff
  • Posts: 144
  • Joined: Jan 09, 2019
|
#63276
Hi brittanychanel!

I think you may be more on track than you realize, and the issue may be in missing the EXCEPT in the question stem. As a 'Must Be True EXCEPT' the correct answer is the only one that is not necessarily true, or in other words, could be false.

We know that the first three slots must be occupied by H, J and Q in that order, but the subsequent branch into both L and P after Q make it impossible to tell which later players must go to which spots. We can, however, deduce where players are NOT allowed to be.

Since the split into fourth place is left for only either L or P, it Must Be True that V is not in fourth place, so C would be wrong in this MBT EXCEPT question for that reason.

L could be either fourth or fifth in this Local question, so the correct response of B which places L in fourth could be false.


I hope that helps!

Happy Sunday :)

Get the most out of your LSAT Prep Plus subscription.

Analyze and track your performance with our Testing and Analytics Package.